ดูหนึ่งข้อความ
  #8  
Old 04 มกราคม 2016, 01:19
Thgx0312555's Avatar
Thgx0312555 Thgx0312555 ไม่อยู่ในระบบ
กระบี่ประสานใจ
 
วันที่สมัครสมาชิก: 12 สิงหาคม 2011
ข้อความ: 885
Thgx0312555 is on a distinguished road
Default

The case prime $a=2$ is unusable since $4 \nmid 3!$
however, the problem only state for $n \ge 4$

ดังนั้นวิธีแบบ inductive แบบนี้ก็ไม่ควรใช้ครับ เพราะมีข้อยกเว้นอยู่กรณีหนึ่ง

จริงๆกรณี composite มีวิธีที่ง่ายกว่านี้ครับ
่Consider $1 \le l < lm^2 <n+1$
__________________
----/---~Alice~ จงรับรู้ไว้ ชื่อแห่งสีสันหนึ่งเดียวที่แสดงผล
---/---- ~Blue~ นี่คือ สีแห่งความหลังอันกว้างใหญ่ของเว็บบอร์ดนี้

04 มกราคม 2016 01:20 : ข้อความนี้ถูกแก้ไขแล้ว 2 ครั้ง, ครั้งล่าสุดโดยคุณ Thgx0312555
ตอบพร้อมอ้างอิงข้อความนี้